Med Surg Test 5 Ch. 52-55 &46

¡Supera tus tareas y exámenes ahora con Quizwiz!

32. Which of the following would the nurse expect to be done to assess the size of the prostate? A. Digital rectal examination B. Transillumination C. Pelvic examination D. Bladder percussion

ANS: A Rationale: A digital rectal examination is performed to assess the prostate for size as well as evidence of tumor. Transillumination is used to determine the density of scrotal tissue. Pelvic examination is performed to evaluate female reproductive structures. Bladder percussion reveals information about the status of the bladder.

22. The nurse is caring for a client hospitalized for pelvic inflammatory disease (PID) who is discovered to have a ruptured pelvic abscess. Which nursing action is the priority at this time? A. Prepare the client for surgery. B. Administer antibiotic therapy as prescribed. C. Provide intravenous fluid replacement. D. Encourage bed rest.

ANS: A Rationale: A ruptured pelvic abscess requires emergency surgery. Administering antibiotic therapy as prescribed, providing intravenous fluid replacement, and encouraging bed rest are all appropriate actions, but they are lower priority for a client with a ruptured pelvic abscess.

14. A hydrocele is suspected in a client with scrotal edema. Which assessment technique would the nurse anticipate to be done first? A. Transillumination of the scrotum B. Palpate for presence of testicle C. Examine for soft tissue swelling D. Assess for collection of blood in the scrotum

ANS: A Rationale: Transillumination, shining a light through the scrotum, will determine if the swelling in the scrotum is caused by serous fluid versus a solid mass. Palpation for testicle is not indicated with a hydrocele. Soft-tissue swelling and/or collection of blood in the scrotum is not indicated with hydrocele.

24. The physician and nurse are completing a physical examination of the male genitalia. Which assessment finding(s) is documented? Select all that apply. A. Inspection for urethral drainage B. Cultures C. Digital rectal examination D. Transillumination E. Palpation of the testes F. Prostate-specific antigen level

ANS: A, C, D, E Rationale: The nurse and physician document the physical assessment including inspection of external genitalia and noting any drainage. A digital rectal examination and transillumination also provides physical assessment data of the status of the prostate gland. Cultures and prostate-specific antigen levels provide diagnostic information but are not part of the physical assessment.

16. The nurse is teaching a client with a history of recurrent vaginal infections about ways to prevent this condition. What should the nurse include in the teaching? Select all that apply. A. Bathe daily. B. Wipe from back to front after bowel movements. C. Avoid douching more than once every 3 days. D. Change from a wet swimsuit as soon as possible. E. Wash hands and devices that are inserted into the vagina.

ANS: A, D, E Rationale: The nurse should teach the client to bathe daily with particular attention to perineal hygiene, wipe from front to back after bowel movements, avoid douching more than once a week, change from a wet bathing suit as soon as possible, and wash hands and devices that are inserted into the vagina.

24. The nurse is caring for a client diagnosed with ductal carcinoma and lymph node involvement. Which diagnostic test, ordered by the physician, does the nurse identify as assessing possible metastasis? A. A complete blood count B. A chest x-ray C. A multiple gated acquisition (MUGA) scan D. A bone density test

ANS: B Rationale: A chest x-ray can identify any tumors present in the lung fields. Lymph nodes containing cancers cells are commonly involved in metastasis, which most frequently spreads to the skeletal and pulmonary systems (in that order). In addition, metastases may be found in the brain, adrenals, and liver. A blood count will not detect metastasis. A bone density study or MUGA scan will not detect metastasis.

15. The nurse is caring for a client who has had diarrhea for 3 days. What major problem(s) associated with severe or prolonged diarrhea should the nurse monitor for when caring for this client? Select all that apply. A. Oral candidiasis B. Dehydration C. Electrolyte imbalances D. Vitamin deficiencies E. Rectal fissures

ANS: B, C, D Rationale: Three major problems associated with severe or prolonged diarrhea include dehydration, electrolyte imbalances, and vitamin deficiencies.

12. The nurse is providing dietary instruction for the client with fibrocystic breast disease. Which of the client's favorite food(s) does the nurse discourage? Select all that apply. A. Lasagna B. Chocolate pudding C. Organ meat D. Cola products E. Popcorn

ANS: B, D Rationale: When instructing the client on appropriate food choices, the nurse instructs the client to avoid caffeine. Caffeine is in products such as chocolate and cola drinks. Lasagna, organ meats, and popcorn do not contain caffeine.

16. What information would the nurse give the client when the client inquires what makes breast milk production? A. Estrogen B. Progesterone C. Prolactin D. Testosterone

ANS: C Rationale: Prolactin promotes the production of breast milk. Estrogen secreted by the ovaries at the onset of puberty causes the development of the mammary system. Progesterone, secreted by the placenta, stimulates the development of alveoli, which secrete the milk. Testosterone does not impact milk production.

29. The nurse is providing shift report related to a client newly received back to the unit from the post anesthesia care unit (PACU). The nurse is stating that the client had breast tissue removed with 7 of 14 lymph nodes, the lining of the chest muscles and pectoralis major and minor muscles removed. The oncoming nurse documents which procedure completed? A. A segmental mastectomy B. A total mastectomy C. A modified radical mastectomy D. A radical mastectomy

ANS: D Rationale: A modified radical mastectomy is a procedure in which the breast, some lymph nodes, the lining over the chest muscles, and the pectoralis minor muscle are removed. A segmental mastectomy is where the tumor and some breast tissue and lymph nodes are removed. A total mastectomy includes only breast tissue. A radical mastectomy removes the breast, axillary lymph nodes, and pectoralis major and minor muscles. Sternal lymph nodes may also be removed with this procedure.

18. A nurse is caring for a client who is being evaluated for a lump found on the lateral aspect of the right breast. The client is anxious and asks the nurse if the next step is to have a mastectomy. Which is the nurse's best response? A. "Yes, breast lumps are abnormal and will need to be removed." B. "The doctor will need a biopsy first to see if the lump is cancerous." C. "The doctor will answer your questions when he or she arrives." D. "Many times, an ultrasound is done to distinguish between a tumor and cyst."

ANS: D Rationale: Ultrasonography is often used with a mammogram or after a breast lump is found to provide additional data on the breast lump. An ultrasound can differentiate if the lump is a fluid filled cyst or other types of breast lesions. This step may be used before a mastectomy is performed or even a biopsy is completed. The nurse is able to provide factual information on common treatment paths, understanding that it is the physician who prescribes the medical care.

14. The nurse is caring for a client who has experienced repeated episodes of vaginitis. Which assessment finding does the nurse explain is the most likely cause? A. Tub baths preferred over shower B. Healthy sex life C. Occasional use of vinegar douches D. Type 2 diabetes mellitus

ANS: D Rationale: Unregulated blood sugars can result in alteration of the vaginal environment and promote the presence of glycogen in vaginal mucus, which supports the growth of microorganisms. Monitoring and controlling blood sugars would be key in a client with diabetes. Tub baths and participation in a healthy sex life are not associated with vaginitis. Frequent douching should be avoided.

16. The clinic nurse is assessing a new male client. Which nursing assessment finding would be cause the nurse to suspect sterility in this client? A. Uncircumcised penis B. Recurrent urinary tract infections (UTI) C. Multiple sex partners D. Mumps at age 15 years

ANS: D Rationale: Viral mumps infection that occurs after puberty can be the cause of orchitis, which may result in testicular atrophy and sterility. Uncircumcised penis, UTIs, and number of sex partners are not indicated with sterility.

12. A client reports loss of interest in sexual relations and discomfort during intercourse. Which is the best response by the nurse? A. "Don't worry, every woman goes through this." B. "These are normal, manageable symptoms of menopause." C. "You need to discuss this with your spouse." D. "Hormone replacement therapy can resolve your symptoms."

NS: B Rationale: Diminished libido and/or dyspareunia are common symptoms associated with perimenopause. Explaining that this can be a normal finding may help to alleviate worries for the client and provide hope for management of the symptoms. The nurse should avoid telling a client "not to worry" or "talk to someone else" because these can negate client feelings and shut down communication. HRT may not be recommended for this client.

20. The nurse is instructing clients at a prenatal class. Which term does the nurse state as the medical name of what is removed during this procedure? A. The prepuce B. The glans C. The shaft D. The corpora cavernosa

ANS: A Rationale: The prepuce or foreskin is removed during circumcision. The prepuce covers the glans. The shaft is tubular and composed of erectile tissue. The corpora cavernosa is a column of erectile tissue.

7. The nurse is teaching a client about the use of a pessary. Which statement by the client indicates the need for additional instruction? A. "A white or yellow vaginal discharge is expected and normal." B. "I need to apply a sterile lubricant to the pessary before reinserting it." C. "After removal, I should wash the pessary with warm soapy water, rinse, and dry it." D. "I should call the physician if I notice any discomfort with the pessary."

ANS: A Rationale: A white or yellow discharge from the vagina is not a normal finding and should be reported to the physician because it may indicate an infection. A sterile lubricant is applied to the pessary before it is reinserted. After removal, the pessary should be washed thoroughly with warm soapy water, followed by rinsing and drying. Discomfort may indicate that a pessary has been inserted incorrectly, it has moved, or that it is causing irritation. These problems should be reported to the physician.

29. A client comes to the outpatient clinic for information on a vasectomy. Which instruction should be included by the nurse? A. Vasectomies should be considered a permanent means of birth control. B. Following surgery, sexual intercourse should be avoided for 3 months. C. Postoperative discomfort can be relieved with use of warm sitz baths. D. Vasectomy can result in permanent impotence.

ANS: A Rationale: Although attempts to reverse a vasectomy have proven successful, a vasectomy should be considered a permanent means of birth control. Scar formation and/or sperm leakage can interfere with restoration of reproductive effectiveness. Sexual intercourse can be resumed when it is comfortable to do so, usually in about 1 week. Postoperative discomfort should be controlled with application of cold to reduce swelling or use of mild analgesics. On occasion, clients have complained of impotence, but vasectomies have no effect on erection or ejaculation.

11. The nurse is caring for a breastfeeding client who has developed a breast abscess. When instructing on the cause of abscess formation, which organism is the most common cause? A. Staphylococcus aureus B. Escherichia coli C. Pseudomonas aeruginos D. Enterococcus faecalis

ANS: A Rationale: An abscess in the breast is most frequently a complication of mastitis. The most common cause is Staphylococcus aureus (S. aureus).

22. A client has developed an anorectal abscess. Which client is likely at risk for the development of this type of abscess? A. A client with Crohn disease B. A client with hemorrhoids C. A client with colon cancer D. A client with diverticulosis

ANS: A Rationale: An anorectal abscess is common in clients with Crohn disease. The other disorders do not predispose the client to risk for anorectal abscess.

8. A client with breast cancer is scheduled to undergo chemotherapy with aromatase inhibitors. Which rationale does the nurse give for the use of this group of drugs? A. They prevent prehormones from becoming estrogen, blocking the tumor's ability to use it. B. They block progesterone-dependent tumors from growing. C. They attach to endogenous protein receptors to slow the growth of cancerous cells. D. They stimulate the immune system to attack a protein common in many tumors.

ANS: A Rationale: Aromatase inhibitors prevent prehormones from becoming estrogen thereby interfering with the ability of hormone-sensitive tumors to use estrogen for growth. Antiprogestin drug, such as mifepristone, blocks progesterone-dependent breast cancers. The monoclonal antibody, trastuzumab attaches to protein receptors to slow the growth of cancer cells. A breast cancer vaccine is under investigation in Italy. This vaccine stimulates the immune system to attack a protein called mammaglobin-A, which is found in 80% of breast cancer tumors.

22. The nurse is caring for a client with benign prostatic hyperplasia (BPH). Which assessment finding provides the best indication of urinary retention? A. Frequency B. Urgency C. Hesitancy D. Dribbling

ANS: A Rationale: As residual urine accumulates, the client has an urge to void more often. Urgency, hesitancy, and dribbling are all urinary symptoms associated with BPH but not specific to urinary retention.

3. The nurse is assisting a 30-year-old client in scheduling an appointment for a cervical biopsy. The nurse would recommend that the client schedule the testing at which time? A. 1 week after completion of the client's menstrual cycle B. 2 weeks before the onset of the client's menstrual cycle C. 3 weeks after the onset of the client's menstrual cycle D. 2 weeks after the cessation of the client's menstrual cycle

ANS: A Rationale: Because the client is premenopausal, the nurse would recommend scheduling the cervical biopsy for 1 week after the end of a menstrual period, when the cervix is least vascular. Scheduling the test for any other time would be inappropriate.

1. The nurse is providing care to a client who has had surgery as treatment for breast cancer. The nurse would be alert for the development of which condition? A. Lymphedema B. Fibrocystic breast disease C. Fibroadenoma D. Breast abscess

ANS: A Rationale: Lymphedema occurs in some women after breast cancer surgery. It causes disfigurement and increases the lifetime potential for infection and poor healing. Fibrocystic breast disease and fibroadenoma are two benign breast conditions that occur usually in premenopausal woman. Breast abscess is the infectious and inflammatory breast condition that is common among breast-feeding mothers.

28. A 50-year-old is diagnosed with stage II prostate cancer. The client is upset and says, "I would rather die than have any surgery." Which is the best response by the nurse? A. "What concerns you most about having surgery?" B. "Your surgeon has performed this surgery many times." C. "How does your family feel about this decision?" D. "This surgery can cure you of cancer."

ANS: A Rationale: Being sympathetic and encouraging the client to express concerns is a therapeutic response. This client may have concerns about the complications associated with the surgery and/or need additional information on newest techniques that improve outcomes. Explaining that the surgeon is competent does not address the concerns of this client. Family feelings are secondary to the concerns or the client. A cure of cancer is never a guarantee with any surgery.

3. A nurse is describing to a client the advantages and disadvantages associated with tamoxifen therapy as a means of breast cancer prevention. The nurse would identify that the drug has a beneficial effect on which factor? A. Bone mineral density B. High density lipoprotein levels C. Risk for endometrial cancer D. Cataract formation

ANS: A Rationale: Besides reducing the risk of breast cancer, tamoxifen preserves bone mineral density, thus preventing osteoporosis. It also lowers the low-density lipoprotein cholesterol levels, although it is still unknown if it decreases the incidence of myocardial infarction in women. Tamoxifen can have detrimental effects. It increases the incidences of endometrial cancer, deep vein thrombosis, pulmonary embolism, and cataracts.

19. The nurse is caring for a client who is diagnosed with an estrogen-sensitive tumor and has undergone a mastectomy. When instructing the client on potential medications, which medication(s) are stressed? A. Tamoxifen (Nolvadex) and aromatase inhibitors (Femara) B. Paclitaxel (Taxol) and doxorubicin (Adriamycin) C. Mifepristone (RU486) D. Testolactone (Teslac)

ANS: A Rationale: Clinical trials suggest that clients with an estrogen-sensitive tumor may benefit from tamoxifen (Nolvadex) or an aromatase inhibitor such asletrozole (Femara) for 5 years or more following a mastectomy. These medications alone or in combinations are found to be effective for preventing breast cancer reoccurrence and metastatic disease.

4. The instructor is teaching a group of students about Crohn disease and antidiarrheal agents. The instructor determines that the teaching was effective when the students identify which of the following as an example of an opiate-based antidiarrheal agent? A. Diphenoxylate with atropine B. Bismuth subsalicylate C. Kaolin and pectin D. Bisacodyl

ANS: A Rationale: Diphenoxylate with atropine is an example of an opiate-based antidiarrheal agent. Bismuth subsalicylate and kaolin and pectin are examples of absorbent antidiarrheal agents. Bisacodyl is a chemical stimulant laxative.

24. The client with benign prostatic hyperplasia (BPH) is considering use of medication in the management of symptoms. The nurse explains that which drug reduces the size of the prostate without lowering circulating levels of testosterone? A. Finasteride (Proscar) B. Tamsulosin (Flomax) C. Terazosin (Hytrin)D. Oxybutynin chloride (Ditropan)

ANS: A Rationale: Finasteride (Proscar) inhibits the conversion of testosterone, depriving the gland of dihydrotestosterone (more potent type of testosterone), which stimulates prostatic growth. Tamsulosin (Flomax) and terazosin (Hytrin) work by reducing the tone of smooth muscle in the bladder neck and prostate gland but have little effect on reducing prostate size. Oxybutynin chloride (Ditropan) is an antimuscarinic, antispasmodic drug used for treatment of overactive bladder.

28. The nurse is preparing the female client for a pelvic examination. Into which position will the nurse assist the client? A. Lithotomy position B. Supine position C. Trendelenburg position D. Lateral recumbent

ANS: A Rationale: For a pelvic examination, the nurse is most correct to assist the client to the lithotomy position. In this position, the client lies on the back with the legs bent and spread. Many times, the feet are in lithotomy stirrups. The supine position is flat on the back. The Trendelenburg position provides the feet higher than the head at a 15° to 30° angle. The lateral recumbent position is a side-lying position.

2. Which of the following would a nurse include in a teaching plan for a client with benign prostatic hyperplasia who is not yet a candidate for surgery? A. Maintaining optimal bladder emptying B. Using appropriate coping to allay anxiety C. Performing deep breathing exercises periodically D. Doing leg exercises at least daily

ANS: A Rationale: For the client with benign prostatic hyperplasia who is not yet a candidate for surgery, the nurse would teach a client how to maintain optimal bladder emptying. The surgical client requires support and information to allay anxiety. The nurse teaches deep breathing and leg exercises for the client who is to have surgery.

9. A client tells the nurse, "I am not having normal bowel movements." When differentiating between what are normal and abnormal bowel habits, what indicators are the most important? A. The consistency of stool and comfort when passing stool B. That the client has a bowel movement daily C. That the stool is formed and soft D. The client is able to fully evacuate with each bowel movement

ANS: A Rationale: In differentiating normal from abnormal, the consistency of stools and the comfort with which a person passes them are more reliable indicators than is the frequency of bowel elimination. People differ greatly in their bowel habits and normal bowel patterns range from three bowel movements per day to three bowel movements per week. It is important for the stool to be soft to pass without pain. The client may not be able to fully evacuate with a bowel movement; it may take time.

3. The nurse is interviewing a client with internal hemorrhoids. What would the nurse expect the client to report? A. Rectal bleeding B. Pain C. Itching D. Soreness

ANS: A Rationale: Internal hemorrhoids cause bleeding but are less likely to cause pain, unless they protrude through the anus. External hemorrhoids may cause few symptoms, or they can produce pain, itching, and soreness of the anal area.

10. The nurse is caring for client who has had a vasectomy. Which would be most important to include in a teaching plan for this client? A. Using a reliable method of contraception for several weeks. B. Applying warm compresses to the scrotum for the first 24 hours C. Taking a prescribed opioid analgesic for pain relief D. Resuming sexual activity in 24 to 48 hours

ANS: A Rationale: It may take several weeks or more after surgery before the ejaculatory fluid is free of sperm, and the client is informed to use a reliable method of contraception until sperm no longer are present. The client should apply ice packs to the scrotum to reduce swelling and use a mild analgesic such as aspirin or acetaminophen for pain relief. The client typically can resume sexual activity when comfortable, usually in 1 week.

1. Which recommendation would a nurse advocate during infancy and childhood to help reduce potential adult complications such as orchitis? A. Ensure immunizations against infectious diseases such as mumps. B. Engage in activities and exercises that minimize heavy lifting. C. Encourage the consumption of foods that are rich in fat and starch. D. Urge the limited intake of foods and fluids containing caffeine.

ANS: A Rationale: Nurses should advocate for infant and childhood immunizations against infectious diseases such as mumps to reduce potential adult complications such as orchitis. Minimizing activities involving heavy lifting or urging limited intake of caffeine have no effect on the potential for adult complications such as orchitis. The nurse should encourage foods that are low in fat and starch; however, this also would have no effect on the potential for orchitis.

19. The nurse is reviewing a client's medication and health history. Which assessment finding would cause the nurse to question the client's new prescription for the phosphodiesterase inhibitor sildenafil? A. Use of nitrates B. History of hypertension C. Type 2 diabetes D. Use of diuretics

ANS: A Rationale: PDE5 inhibitors facilitate penile erection by producing smooth muscle relaxation in the corpora cavernosa via vasodilation of the blood vessels. If a client is using nitrates (which also exhibit vasodilation) hypotension is likely to occur. History of hypertension and diabetes does not interfere with the administration of PDE5 inhibitors. Use of diuretics is not significant.

1. Which of the following is an early symptom of vulvar cancer? A. Pruritus with genital burning B. Fever accompanied by chills C. Severe abdominal pain D. Dyspareunia

ANS: A Rationale: Pruritus and genital burning are the most frequent early symptoms of vulvar cancer, followed by a bloody discharge from the vagina. Abdominal pain can be a result of formation of cysts in the ovary. Dyspareunia and fever accompanied by chills are not the early symptoms of vulvar cancer.

20. The nurse is talking with a group of clients who are older than age 50 years about the recognition of colon cancer to access early intervention. What should the nurse inform the clients to report immediately to their primary care providers? A. Change in bowel habits B. Excess gas C. Daily bowel movements D. Abdominal cramping when having a bowel movement

ANS: A Rationale: The chief characteristic of cancer of the colon is a change in bowel habits, such as alternating constipation and diarrhea. Excess gas, daily bowel movements, and abdominal cramping when having a bowel movement are not indicators of colon cancer.

23. The nurse is caring for a client scheduled for a transrectal ultrasonography. Which instruction is essential in obtaining an accurate test? A. Self-administer an enema prior to arrival. B. Take a low-dose sedative prior to arrival. C. Avoid fluids after midnight. D. Empty the bladder prior to the exam.

ANS: A Rationale: The client will need to have an enema to remove stool from the rectum so that the ultrasound can penetrate through the tissues to visualize structures of the prostate. Typically, a sedative is not needed as the client uses normal relaxation techniques. Urine status is not the focus of the exam requiring that fluids be withheld. For the client's comfort, the nurse may encourage emptying the bladder before the exam.

23. The nurse is caring for four clients in an oncology unit. Following report,, which client would the nurse identify as having the most life-threatening diagnosis? A. A client with stage IV cancer receiving palliative care B. A client diagnosed with a stage one tumor with no lymph node involvement C. A client receiving chemotherapy and radiation with a low blood count D. A client with an estrogen receptor-positive tumor receiving oral chemotherapy

ANS: A Rationale: The client with stage IV cancer, indicating metastasis, and receiving palliative care, which includes comfort measures, is in highest life-threatening condition. A client with a stage I tumor and no lymph node involvement has a good prognosis as the cancer has been caught at an early stage. A low blood count is common when the client is receiving chemotherapy. It is common to have chemotherapy and radiation as a treatment regimen. Clients have experienced good prognosis with estrogen receptor positive tumor treatment of oral chemotherapy.

8. The expert nurse is assisting a novice nurse insert a Foley catheter. The novice nurse has tried unsuccessfully to insert the catheter, and the expert nurse is providing verbal guidance while spreading which area to reveal the urethral opening? A. Labia majora and minora B. Meatus C. Clitoris D. Fourchette

ANS: A Rationale: The labia majora and minora are a portion of the external genitalia and when parted reveals the urethral opening. The meatus refers to an opening or passage. The clitoris is sensitive erectile tissue considered the site of sexual pleasure. The fourchette is the area beneath the vaginal opening at the base of the labia majora.

25. The nurse is caring for a client who has been diagnosed with a breast lump. The physician states that studies indicate a cancerous tumor in situ. The client is anxious and wants to know what may come next. The nurse is most correct to support physician instruction of which of the following? A. "Since your tumor appears localized, removal of the tumor is indicated." B. "A mastectomy is recommended for all clients in this situation." C. "The next step is just as the physician described. We can have the physician talk to you again." D. "Chemotherapy and radiation are typical treatments prescribed by the oncologist."

ANS: A Rationale: The nurse is correct to restate the information that the physician has just discussed. Many times, anxious clients have a difficult time understanding information. The nurse realizes that a tumor that is in situ has developed in a localized area without invading the surrounding tissue. This criterion would allow the removal of tissue from only that area, not an entire mastectomy. When the client asks for the next step, chemotherapy and radiation, as indicated, would be initiated after removal of the tumor.

14. A client reports taking a stimulant laxative in order to be able to have a bowel movement daily. What should the nurse inform the client about taking a stimulant laxative? A. They can be habit forming and will require increasing doses to be effective. B. If the client is drinking 8 glasses of water per day, it is all right to continue taking them. C. The laxative is safe to take with other medication the client is taking. D. The client should take a fiber supplement along with the stimulant laxative.

ANS: A Rationale: The nurse should discourage self-treatment with daily or frequent enemas or laxatives. Chronic use of such products causes natural bowel function to be sluggish. In addition, laxatives continuing stimulants can be habit forming, requiring continued use in increasing doses. Although the nurse should encourage the client to have adequate fluid intake, laxative use should not be encouraged. The laxative may interact with other medications the client is taking and may cause a decrease in absorption. A fiber supplement may be taken alone but should not be taken with a stimulant laxative.

20. A client who wishes to preserve childbearing ability asks the nurse to explain how taking oral contraceptives will work in the management of endometriosis. Which is the best response by the nurse? A. "Symptoms of endometriosis are increased during normal menstrual cycle." B. "Contraceptives will allow blood to be diverted to the peritoneal cavity." C. "Trapping blood causes less pain or discomfort for clients with endometriosis." D. "Endometriosis is usually cured with surgical menopause."

ANS: A Rationale: The use of estrogen-progestin contraceptives keeps the client in a non-bleeding phase of the menstrual cycle, therefore decreasing ectopic tissue from shedding and causing extra uterine bleeding. Blood that is trapped in the peritoneal cavity causes more pain as adhesions form. Endometriosis is cured by natural or surgical menopause but can be medically managed for periods of time with the use of oral contraceptives.

1. Which of the following would be most important for the nurse to include to ensure a thorough baseline history related to the reproductive system? A. Age of menarche B. Nutritional history C. Exercise and fitness routine D. Cultural history

ANS: A Rationale: To ensure a thorough baseline history, the nurse obtains information on the client's general health and family history, age of menarche, and menstrual history. Although the client's nutritional history, exercise and fitness routines, and cultural history are important, they are part of the overall client health history and not specific to the reproductive system

15. The nurse is providing an educational presentation on toxic shock syndrome (TSS) to a group of adolescents. Which preventive measure does the nurse focus on for this group? A. If using tampons, change frequently. B. Do not use vaginal devices for birth control. C. Toxin is associated with multiple sex partners. D. Use good personal hygiene.

ANS: A Rationale: Toxic shock syndrome is often associated with the use of superabsorbent tampons that are not changed frequently. The infection can also be introduced through unclean birth control devices or keeping the device in place too long after use. Good personal hygiene and safe sexual practices should also be addressed but they are not specific to TSS.

11. The nurse is assessing a client for fecal impaction, and when inserting a lubricated, gloved finger, the stool feels like small rocks. What does the nurse document this finding as? A. Scybala B. Hard stool C. Fecal Impaction D. Obstruction

ANS: A Rationale: When a practitioner inserts a gloved and lubricated finger in the rectum, the stool may feel like small rocks, a condition referred to as scybala. The client may have hard stool or be impacted but the correct terminology to be documented is scybala. A fecal obstruction is not always able to be determined on digital examination and will require an x-ray.

24. Which risk factor(s) supports the need for ovarian cancer screening? Select all that apply. A. Carrying genetic mutation for the disease B. Being nulliparous C. Family history of lung cancer D. Breastfeeding E. Use of oral contraceptives for 5 years or more F. Multiple full-term pregnancies

ANS: A, B Rationale: Woman who are nulliparous, have a family history of ovarian cancer, and/or carry a genetic mutation for it tend to develop ovarian cancer more often than others. Preventive measures recommended to lower the risk of ovarian cancer include having at least 2 full-term pregnancies, breastfeeding after pregnancies, and using oral contraceptives for 5 years or more. History of lung cancer is not a risk factor.

10. The nurse is caring for a client with polycystic ovarian syndrome. On which associated problem(s) will the nurse focus teaching? Select all that apply. A. Hypertension B. Hyperlipidemia C. Infertility D. Absence of body hair E. Weight loss F. Decreased Insulin

ANS: A, B, C Rationale: Polycystic ovarian syndrome is associated with multiple endocrine abnormalities such as abnormal lipid levels, hypertension, and difficulty with conceiving due to interference of ovulation. Other symptoms include excessive body hair, weight gain, and overproduction and inefficient use of insulin.

15. The client with prostatitis presents with low back pain, dysuria, and unusual sensation following ejaculation. Which treatment(s) does the nurse anticipate being prescribed? Select all that apply. A. Analgesics B. Sitz baths C. Antibiotics D. Abstinence E. Treatment of sexual partner

ANS: A, B, C, E Rationale: Prostatitis is an inflammation of the prostate gland that is most often caused by microorganisms. Treatment consists of up to 30 days of antibiotic therapy, mild analgesics, and sitz baths. Sexual partners also need to be treated. Regular drainage of the prostate gland through masturbation or intercourse can be helpful in decreasing the inflammation and discomfort.

18. The nurse is working at an oncologist's office specializing in breast cancer. The nurse cares for individuals of varied ages, backgrounds, and stages of breast cancer. Which criteria are considered a priority when determining adjunct treatment in addition to surgery? Select all that apply. A. Stage of tumor B. Cosmetic consequence C. Presence of metastasis D. Age E. Gender F. Sensitivity to estrogen

ANS: A, C, D, F Rationale: When working in an oncologists' office, the nurse is correct to identify that the stage of tumor and presence of metastasis help to guide treatment. Other factors include age of client and sensitivity to estrogen. Cosmetic consequence is not considered a priority when considering lifesaving treatment options. Many times, reconstruction is a possibility. Gender is not a consideration because treatment options are very similar.

8. A female client is diagnosed with carcinoma in situ of the endometrium. The nurse interprets this as which of the following? A. Cancer is confined to the body of the uterus. B. The malignancy is localized. C. Cancer extends outside the uterus. D. The malignancy involves the uterine body and cervix.

ANS: B Rationale: A localized malignancy is referred to as carcinoma in situ. Stage I endometrial cancer is confined to the body (corpus) of the uterus, stage II involves the corpus and cervix, and stage III extends outside the uterus but not the true pelvis.

4. A client is scheduled to undergo a procedure to lift breasts that have drooped because of significant weight loss. The nurse identifies this procedure as which of the following? A. Reduction mammoplasty B. Mastopexy C. Autogenous breast reconstruction D. Mastectomy

ANS: B Rationale: A mastopexy is a procedure that involves lifting a breast. This procedure is similar to a reduction mammoplasty (removal of glandular breast tissue, fat, and skin bilaterally to decrease the size of large pendulous breasts), although the incision and scar line are smaller and the recovery time is shorter. Autogenous breast reconstruction involves the harvesting of the client's tissue from the rectus abdominis muscle to create a new breast. A mastectomy is a removal of a breast.

2. An instructor is preparing a teaching plan for a group of students about reproductive structures and sexual arousal. Which female reproductive structure would the instructor include? A. The mons pubis B. The clitoris C. Labia minora D. Labia majora

ANS: B Rationale: At the superior junction of the labia, there is a fleshy protrusion of tissue called the clitoris. The clitoris is an erectile tissue that enlarges and becomes sensitive when stimulated by the penis or touching that accompanies sexual foreplay. The mons pubis, labia minora, and labia majora are female reproductive structures but do not play as prominent a role in sexual arousal.

27. The nurse is caring for a client who will be having artificial implants for breast reconstruction. The client is arriving at the physician's office for which procedure to be completed before the surgery can be done? A. Incisional alignment B. Tissue expansion C. Fluid drainage D. Pain control

ANS: B Rationale: Before an implant for breast reconstruction can produce an optimum cosmetic appearance, the skin and tissue on the chest wall are expanded to provide a large enough space to fill and approximate the size of the remaining breast. The other options are not correct.

17. The nurse is collecting assessment data on a client who is reporting a vaginal discharge that is cottage cheese-like in appearance. Which pathogen is the most likely cause for this symptom? A. Gonococci B. Candida albicans C. Trichomonas vaginalis D. Gardnerella vaginalis

ANS: B Rationale: Candida albicans is a yeast infection that presents with a thick, curdy white discharge. Gonococcus is the organism that causes gonorrhea and presents with a yellow, mucopurulent discharge. Trichomonas vaginalis presents with a foamy, white, foul-smelling discharge and Gardnerella vaginalis is a watery, fishy-smelling discharge.

23. Following a colposcopy, the confirmation of in situ carcinoma of the cervix has been determined. Which comment by the client indicates an appropriate understanding of the diagnosis? A. "I will not need any further treatment." B. "The cancer has not spread." C. "I will need surgery and chemotherapy to increase my odds for survival." D. "I can wait until I have finished having babies to seek treatment."

ANS: B Rationale: Carcinoma in situ means the cancer has not left the original site and therefore has not invaded other tissues. Further treatment is required, which usually consists of cryosurgery or hysterectomy. Surgery with adjuvant chemotherapy is usually reserved for cancers that are more advanced. Even though cervical cancers tend to be slower growing, treatment should not be delayed.

17. Which nursing assessment finding is most significant in determining the plan of care in a client with erectile dysfunction? A. Age B. Medication use C. Sexual history D. Undescended testicle

ANS: B Rationale: Certain medications such as antihypertensive drugs, antidepressants, narcotics, etc. can cause sexual dysfunction in men. Impotence is not a normal part of aging. Undescended testicle is not indicative of ED. Sexual history is not indicative of ED.

15. A nurse is caring for a client following a culdoscopy. Which assessment data would the licensed practical nurse report to the registered nurse? A. Pain level of 7 on the 0-to-10 pain scale B. Skin, cold; pulse,110 beat/minute; blood pressure, 104/64 mm Hg C. Dark drainage noted on surgical pad D. Fleshy smell noted to perineal region

ANS: B Rationale: Culdoscopy is performed to visualize the uterus, broad ligaments, and fallopian tubes. An endoscope is inserted through a small incision made in the posterior vaginal wall. Afterward, the nurse must observe for signs of internal bleeding and shock. A client with cold and clammy skin, a rapid pulse, and low blood pressure indicates shock and the need to consult the registered nurse and even the physician. All of the other symptoms are able to be addressed by the licensed practical nurse. The client is to be medicated. Dark drainage and a fleshy smell to the perineal area are normal for old surgical drainage.

9. A client tells the nurse that she is struggling with symptoms of irritability and "feeling crazy" around the same time each month. Which assessment data would provide the most important information needed in the development of a treatment plan for this client? A. Reproductive history B. Menstrual diary C. Vital signs D. Dietary assessment

ANS: B Rationale: Diagnosis of premenstrual disorder and/or premenstrual dysphoric disorder is based on data recorded in a menstrual diary over the time period lasting at least 2 months or more. Symptoms are absent during the period between onset of menstruation and ovulation as recorded in the diary. Reproductive history, vital signs, and dietary assessment are not significant in the diagnosis of the PMS or PMDD.

14. While the nurse is providing education for a client with a family history of breast cancer, the client asks, "What is the most common type of breast cancer?" What is the correct response from the nurse? A. Inflammatory breast cancer B. Ductal carcinoma C. Lobular carcinoma D. Medullary carcinoma

ANS: B Rationale: Ductal carcinoma makes up 70% to 80% of cases of breast malignancy. Inflammatory breast cancer is 1% to 5%, lobular carcinoma is 5% to 10%, and medullary carcinoma is the rarest among those listed.

22. The nurse is obtaining a history from a male client who states having difficulty achieving and sustaining an erection. When reviewing the medication history, which medication classification does the nurse anticipate? A. Bronchodilators B. Antihypertensives C. Cardiac dysrhythmics D. Antibiotics

ANS: B Rationale: Due to the therapeutic effect of antihypertensive medications decreasing blood pressure, the nurse is correct to anticipate a side effect of a male client having difficulty achieving or sustaining an erection. The other medication classifications do not typically have an effect on the male reproductive system.

30. During a physical examination, the nurse inspects the external genitalia of a male client. Which of the following would the nurse identify as an abnormality? A. Absence of urethral discharge B. Skin lesions C. Prepuce covering the glans penis D. Slightly pendulous scrotum

ANS: B Rationale: During a physical examination, the nurse inspects the external genitalia, looking for abnormalities such as skin lesions and urethral discharge. Prepuce covering the glans penis is a normal finding in an uncircumcised male. The scrotum is normally pendulous, and with increasing age, it becomes more pendulous.

17. The nurse is completing community education when asked by a client without health insurance why a mammogram is needed if the women are completing breast self-examinations at home. The nurse is most correct to respond stating which of the following? A. "Mammograms are just better." B. "Mammograms can detect cysts or tumors too small to palpate." C. "Mammograms provide a baseline that shows changes in breast tissue." D. "Mammograms provide a reassurance that the breast is free of cancer."

ANS: B Rationale: During community instruction, it is most important to relay facts in a clear, factual, and nonjudgmental way. Stating the need for mammograms to detect small cysts or tumors is clear and factual. Stating the mammograms are "better" is vague and provides no better understanding. Although a mammogram does provide a baseline and reassurance of breast health, the most clear and factual reason why the mammogram is completed is its ability to detect cancerous tumors at an early stage.

23. The nurse is caring for a client with intussusception of the bowel. What does the nurse understand occurs with this disorder? A. The bowel twists and turns itself and obstructs the intestinal lumen. B. One part of the intestine telescopes into another portion of the intestine. C. The bowel protrudes through a weakened area in the abdominal wall. D. A loop of intestine adheres to an area that is healing slowly after surgery.

ANS: B Rationale: In intussusception of the bowel, one part of the intestine telescopes into another portion of the intestine. When the bowel twists and turns itself and obstructs the intestinal lumen, this is known as a volvulus. A hernia is when the bowel protrudes through a weakened area in the abdominal wall. An adhesion is a loop of intestine that adheres to an area that is healing slowly after surgery.

5. A client with erectile dysfunction who had a penile implant inserted has been taught how to identify malfunction of the device. Which characteristic named by the client, indicative of malfunction, would show the nurse that the client has understood the teaching? A. Erosion of penile or urethral tissue B. Underinflation or bulging of the cylinders during inflation C. Erosion of scrotal, bowel, or bladder tissue D. Migration of the cylinders, pump, or reservoir from their intended location

ANS: B Rationale: Malfunction of the device is characterized by the underinflation, bulging of the cylinders during inflation, and a loss of fluid from the implant. Erosion of penile or urethral tissue occurs due to a midsized implant, pressure, or friction of the implanted cylinders. Erosion of scrotal, bowel, or bladder tissue occurs if an implant with a fluid reservoir is used. Migration of the cylinders, pump, or reservoir from their intended location is accompanied by pain, tenderness, and dysfunction of components that are part of the device.

25. The client states to the nurse that he is very anxious about having prostate cancer ever since his prostate-specific antigen (PSA) test came back elevated. The client asks, "Which diagnostic test produces definitive results if cancer is present?" The nurse is most correct to state which of the following? A. Transrectal ultrasonography B. Tissue biopsy C. Tumor marker studies D. Digital rectal exam

ANS: B Rationale: Obtaining an actual piece of the tissue and analyzing it for cancer is a definitive test when cancer is found. A transrectal ultrasonography is a test to view the prostate gland from different angles. This test provides additional data on the status of the prostate gland. The tumor marker studies include the prostate-specific antigen (PSA) level. This antigen indicates a potential problem but is not definitive. PSA elevations have been noted for reasons other than cancer. A digital rectal exam provided data on the shape, size, and texture of the prostate gland.

19. Which assessment finding will most likely influence the treatment regimen selected by a client with endometriosis? A. Presence of pain B. Family planning C. Dysmenorrhea D. Presence of chocolate cyst

ANS: B Rationale: Selection of a more conservative option may be indicated to preserve the possibility for future childbearing. Pain, dysmenorrhea, and chocolate cysts are all symptoms associated with endometriosis and can influence decision for treatment.

9. A client is diagnosed with stage II cancer of the prostate. The nurse interprets this as indicating which of the following? A. Cancer is small with no symptoms. B. Tumor is confined to the prostate gland. C. Tumor has spread beyond the prostrate. D. Tumor has expanded to the bladder.

ANS: B Rationale: Stage II cancer of the prostate involves a tumor that is larger than stage I but is still confined to the prostate gland; if left untreated, it is more likely to spread beyond the prostate and cause symptoms. Stage I cancer of the prostate involves a small, slow-growing cancer that may never cause symptoms or other health problems. Stage III involves a tumor that has spread beyond the prostate but has not reached the bladder, rectum, lymph nodes, or other organs. Stage IV involves a tumor that has spread to the bladder, rectum, lymph nodes, or distant organs such as the bones.

21. A nurse is employed by a fertility clinic. Diagnostic testing is being completed to determine the source of the male not producing viable sperm to enable ovum fertilization. Which male structures does the nurse consider if the problem is narrowed to sperm survivability? A. Bartholin glands B. Prostate gland C. Cowper glands D. Vas deferens

ANS: B Rationale: The accessory structure of the prostate gland contains secretory cells that produce alkaline fluid. The prostatic fluid mixes with sperm. The alkalinity of the prostatic fluid neutralizes the acidic metabolic wastes released by the sperm and counteracts the acid pH within the vagina to ensure mass survival of the sperm. The Cowper glands add lubrication for penetration of the vagina.

9. The nurse is obtaining a history from a client who states recurrent breast tenderness. The client inquires if there is something wrong with her breasts. Which question by the nurse is a priority? A. "Do you have a history of breast disorders and cancer in the family?" B. "Does the tenderness occur around the same time each month?" C. "Have you pulled a muscle or had any injury to the breast?" D. "When did you first recognize the symptoms?"

ANS: B Rationale: The breasts are part of the female reproductive system, and they respond to the hormonal cycle associated with ovulation, menstruation, and pregnancy. Because the hormonal cycle is monthly, it is best to ask the client if the tenderness occurs at the same time each month. The other options are also important questions to ask.

6. The nurse is teaching a client with constipation to increase dietary fiber intake to 25 g/day. Which recommendation would the nurse include? A. Avoiding bran cereals and beans in the diet B. Adding fiber-rich foods to the diet gradually C. Limiting fluid intake to 5 to 6 glasses per day D. Minimizing activity levels for at least 2 months

ANS: B Rationale: The nurse instructs the client to add fiber-rich foods to the diet gradually to avoid bloating, gas, and diarrhea. It is essential for a client to include bran cereals and beans in the diet because they ease defecation. The nurse also instructs the client to increase fluids to 6 to 8 glasses per day to prevent hard, dry stools. The client should also develop a regular exercise program to increase peristalsis and promote bowel elimination.

7. During a follow-up visit, a female client who underwent a mastectomy presents with an infection that requires an antibiotic. The client reports having been doing some gardening. What further instruction and reinforcement of teaching should the nurse provide? A. Avoid working in the garden or yard altogether. B. Wear gloves and protective clothing to avoid any injuries. C. Increase the frequency of follow-up visits if she does work. D. Avoid household chores for at least 6 to 9 months.

ANS: B Rationale: The nurse should recommend that the client wear gloves when doing backyard work or housework to prevent injuries that may heal slowly or become infected. Working, whether it is in the backyard or doing some household chores, can be helpful in promoting feelings of usefulness, thereby enhancing the client's coping abilities and self-esteem. The client could be advised to follow up more frequently, however, this would not help prevent any untoward injury.

15. A suspicious breast lump is noted on a mammogram. The client asks the nurse which diagnostic test confirms if the lump is cancerous or benign. Which response by the nurse is most correct? A. An ultrasound B. A biopsy C. A magnetic resonance imaging (MRI) D. A clinical breast exam

ANS: B Rationale: To confirm whether a breast lump is cancerous or benign, a tissue sample must be obtained to examine the cells. Although an ultrasound, MRI, and clinical breast exam provide data on the characteristics of the lump, only examining the tissue can specifically identify if and what type of cancer is present.

2. A client with a painless, firm, nontender lump in the breast is being scheduled for diagnostic testing. Which test would the nurse expect the physician to order? A. Standard mammogram B. Ultrasound C. Excisional biopsy D. Culture of discharge

ANS: B Rationale: Ultrasound can reveal physical characteristics unique to a fibroadenoma versus malignant mass with a higher degree of accuracy than standard mammography. In the case of very young women—an atypical age for breast cancer— an excisional biopsy is performed only if the mass changes or becomes larger. If the mass is detected in a woman with a higher risk for developing breast cancer, such as one with a family history or of an older age, a biopsy is performed to confirm that the tissue is indeed benign. There is no discharge to culture.

26. The nurse is instructing a couple in the early stages of infertility testing who has just found that the male has a slightly low sperm count. Which easy change can a nurse suggest to a male client? A. Avoid riding on motorcycles. B. Wear loose fitting underwear instead of tight ones. C. Take a general multivitamin. D. Increase sexual intercourse to every day.

ANS: B Rationale: Wearing loose fitting underwear allows the scrotum to pull the testes closer to the body to maintain proper temperature for sperm production or allow the testes to be farther from the body. Because the testes cannot produce viable sperm when temperatures are at or above body temperature, their location within the scrotal sac ensures optimal conditions. There is no evidence that the client cannot ride motorcycles. Taking a general multivitamin is encouraged for overall health. Daily sexual intercourse is not encouraged because it is most important to have sufficient sperm present during a female's ovulation period.

4. The nurse is providing teaching to a client who is scheduled for a mammogram. Which of the following would the nurse include as a way to avoid artifacts on the x-ray film? A. "Don't wear any jewelry around your neck on the day of the test." B. "Avoid using a deodorant with aluminum hydroxide." C. "Make sure not to eat or drink for 4 hours before the test." D. "Drink at least 1 quart of water 45 minutes to 1 hour before the test."

ANS: B Rationale: When a mammogram is scheduled, the nurse explains the radiographic procedure and instructs the client to avoid using a deodorant with aluminum hydroxide or body talc on the day of the test to avoid artifacts on the x-ray film. Not wearing jewelry is a precaution for any radiographic test. Fasting is not required before a mammogram. Drinking water before the test is appropriate for an abdominal ultrasound.

20. A client is scheduled for a pump-type implanted penile prosthesis. Which client statement to the nurse would indicate the need for additional teaching? A. "The reservoir is filled with saline solution." B. "The penis will appear longer than usual." C. "Surgery is required for placing the implant." D. "The implant is pumped when sexual activity is desired."

ANS: B Rationale: When a pump-type implant is used, the erect penis tends to be shorter than usual because the saline filled cylinders do not fill the glans portion of the penis. This procedure is performed via surgical approach. The implant is pumped causing filling and erection when sexual activity is desired.

11. The nurse is caring for a young adult client who reports excessive menstrual flow with menses lasting 10 days each month. Which treatment option does the nurse anticipate the physician will order for symptomatic relief for this client? A. Dilation and curettage (D and C) B. Hormonal contraceptives C. Endometrial ablation D. Uterine balloon therapy

ANS: B Rationale: When combination hormonal contraceptives are administered, they produce a light menstrual period and are helpful in the management of menorrhagia. Dilation and curettage can provide symptomatic relief, however, the effectiveness is limited. Endometrial ablation and uterine balloon therapy cause the endometrium to slough and are cost-effective but are only used after childbearing has completed.

12. A young client is admitted with torsion of the spermatic cord. Which is the appropriate action to be taken by the nurse? A. Elevate the scrotum. B. Keep the client NPO. C. Monitor vital signs for cardiac changes. D. Avoid the use of analgesics.

ANS: B Rationale: With torsion of the spermatic cord, immediate surgery is necessary to prevent atrophy of the cord and preserve fertility. The client should be placed on NPO status in preparation for surgery. Elevating the scrotum intensifies the pain by increasing the degree of twist. Analgesics are prescribed preoperatively to control pain. Cardiac changes are not indicated unless client history warrants assessment.

11. A client presents with symptoms of menstrual difficulties and infertility. Which hormone is essential in the maintenance of the endometrium? A. Estrogen B. Progesterone C. Testosterone D. Prolactin

ANS: B Rationale: Without a high level of progesterone, the endometrium (uterine lining) degenerates and shreds. Estrogen is a hormone that is essential for menstruation and prepares the uterus for pregnancy. Women produce testosterone, which maintains muscle mass, bone strength, and sex drive. Prolactin is produced by the pituitary gland and stimulates lactation.

25. The surgeon has ordered belladonna and opium (B and O) suppositories for the management of a postoperative prostatectomy client who is receiving continuous bladder irrigation. Which symptom(s) would indicate to the nurse the need for administration of this drug? Select all that apply. A. Increased blood in the catheter tubing B. Slowed or stopped infusion rate C. Increased bladder pressure and pain D. Increased speed of infusion rate E. Signs of urinary retentionF. Increased force of urinary stream

ANS: B, C Rationale: Belladonna and opium suppositories are ordered to reduce bladder spasms that cause increased bladder pressure and pain following a prostatectomy. As spasms occur, the infusion of saline will slow or stop. Increased blood in the catheter tubing is a sign of active bleeding. With an indwelling three-way catheter, urinary retention and increased force of urinary stream are not present.

23. The nurse is teaching a client about the management of benign prostatic hyperplasia (BPH). Which instruction(s) should be included in the teaching? Select all that apply. A. Moderate use of alcohol is useful for bladder relaxation. B. Do not delay the urge to void. C. Low-dose Benadryl will promote restful sleep. D. Prolonged exposure to heat increases bladder spasms. E. Painless hematuria is a common symptom of BPH. Schedule digital rectal exams.

ANS: B, F Rationale: The client should be instructed to void promptly when the urge to empty the bladder is signaled by the stretch receptors in the bladder. Voiding promptly will decrease the risk for urinary retention. Digital rectal exams should be monitored to detect further enlargement of the gland and/or presence of prostatic nodules. Alcohol and antihistamines (e.g., Benadryl) should be avoided in the management of BPH. Exposure to heat and painless hematuria are not significant in the management of BPH. Alcohol and antihistamines interact with many BPH drugs.

27. A client who is diagnosed with stage II prostate cancer asks the nurse if a transurethral resection of the prostate (TURP) can be done for this disorder. Which is the best response by the nurse? A. "A TURP is a viable option. Let's talk to the surgeon." B. "Stage II means the cancer has already spread from the prostate gland." C. "A TURP is a removal of only a portion of the prostate gland." D. "You have the right to choose whichever surgery you feel is best for you."

ANS: C Rationale: A TURP is not a viable option for treatment of prostate cancer and will only scrape or remove the portion of the gland that is causing obstruction of the urethra. Stage II cancer is a tumor that is still confined to the prostate gland and has not metastasized. The surgical treatment of choice is determined by the surgeon and client together and is not based on feelings but rather evidence-based practice and best prognosis and outcome.

21. A client is diagnosed with colon cancer, located in the lower third of the rectum. What does the nurse understand will be the surgical treatment option for this client? A. Colectomy B. Segmental resection C. Abdominoperineal resection D. A low colectomy

ANS: C Rationale: A cancerous mass in the lower third of the rectum will result in an abdominoperineal resection with a wide excision of the rectum and the creation of a sigmoid colostomy. An encapsulated colorectal tumor may be removed without taking away surrounding healthy tissue. This type of tumor, however, may call for partial or complete surgical removal of the colon (colectomy). Occasionally, the tumor causes a partial or complete bowel obstruction. If the tumor is in the colon and upper third of the rectum, a segmental resection is performed. In this procedure, the surgeon removes the cancerous portion of the colon and rejoins the remaining portions of the GI tract to restore normal intestinal continuity.v

27. The client is asking the nurse to explain the typical treatment expected when a class 4 Papanicolaou test is reported. The nurse is most correct to state which of the following? A. No further treatment is considered. B. The test will need to be repeated in 3 months. C. Further evaluation is needed for potential malignancy. D. Chemotherapy is needed and radiation is suggested.

ANS: C Rationale: A class 4 Papanicolaou test (Pap test) is strongly suggestive of malignancy. Further testing is needed to confirm the type and extent of cancer. The nurse would not suggest that no treatment is needed or to wait for 3 months. The nurse would also not suggest a typical treatment without further diagnostic testing.

3. A client with erectile dysfunction has opted to self-administer a urethral suppository to produce an erection. Which agent would be used? A. Sildenafil B. Vardenafil C. Alprostadil D. Papaverine

ANS: C Rationale: Alprostadil is the drug used for self-administration via a urethral suppository. Sildenafil and vardenafil are examples of phosphodiesterase inhibitors that are taken orally. Papaverine with phentolamine is used for self-injection into the corpora cavernosa to achieve an erection.

26. A client with a normal prostate-specific antigen (PSA) level is reporting painful urination and the presence of blood in urine and semen. The nurse understands that the best option for management of this client includes which of the following? A. Wait-and-see approach B. Continue to monitor PSA levels. C. Further evaluation D. Treat the symptoms.

ANS: C Rationale: Although PSA levels of 4 ng/mL or greater is a basis for performing more definitive diagnostic tests, screening measurements of PSA are becoming more controversial. The presence of dysuria, hematuria, and hemospermia are indicative of prostate tumor growth and should be receive further evaluation. The wait-and-see approach is not appropriate if the client is symptomatic. Treating the symptoms without diagnosis could place the client at greater risk for metastasis.

2. A nurse is preparing a presentation for a local community group of older adults about colon cancer. What would the nurse include as the primary characteristic associated with this disorder? A. Abdominal distention B. Frank blood in the stool C. A change in bowel habits D. Abdominal pain

ANS: C Rationale: Although abdominal distention and blood in the stool (frank or occult) may be present, the chief characteristic of cancer of the colon is a change in bowel habits, such as alternating constipation and diarrhea. Abdominal pain is a late sign.

18. The nurse is caring for a client with erectile dysfunction. Because the client has coronary artery disease (CAD), the nurse anticipates the provider will prescribe which medication for ED? A. Sildenafil (Viagra) B. Tadalafil (Cialis) C. Apomorphine (Uprima) D. Vardenafil (Levitra)

ANS: C Rationale: Apomorphine (Uprima), a dopamine agonist, is a possible alternative to phosphodiesterase (PDE5) inhibitors, such as sildenafil (Viagra), tadalafil (Cialis), and vardenafil (Levitra) for the treatment of ED. Uprima is safer for men with CAD and does not cause mass vasodilation of the blood vessels such as the PDE5 inhibitors.

10. The lactation nurse is caring for a mother who is 2 weeks postpartum and has mastitis. Which statement, made by the client, requires instruction and is the probable cause of the mastitis? A. "I feed the baby every 2 hours." B. "I break the baby's suck before pulling the baby off of the breast." C. "I nurse the baby on one breast each feeding." D. "I use a lanolin ointment on my dry nipples."

ANS: C Rationale: Because the client has mastitis, emptying each breast with alternate feedings is important. The baby's suck is strongest at the beginning of the feeding, thus, the mother should alternate the breast that the baby starts nursing from. If not, one breast would not be drained of the milk.

11. A clinic nurse is scheduled to see four male clients. Which assessment finding is most important in determining which client has a higher risk for developing testicular cancer? A. Previous sexually transmitted infection (STI) B. Low sperm count C. Cryptorchidism as an infant D. Family history of cancer

ANS: C Rationale: Caucasian men who have had cryptorchidism as an infant, regardless of whether an orchiopexy was performed, are at higher risk for incidence of testicular cancer. STIs, low sperm count, and family history of general cancers are not indicative of testicular cancer risk.

8. The nurse is obtaining a medication history from a client who is reporting erectile dysfunction. Which medication would the nurse identify as being least likely to contribute to the client's condition? A. Spironolactone B. Cimetidine C. Ibuprofen D. Methyldopa

ANS: C Rationale: Certain medications, such as antihypertensive agents (e.g., methyldopa and spironolactone), antidepressants, narcotics, and cimetidine cause sexual dysfunction in men. Ibuprofen is not associated with causing erectile dysfunction. Cimetidine is a histamine2-receptor antagonist used for gastric ulcers.

6. A client complains of having tender and painful breasts, often feeling multiple lumps within the breast tissue. The nurse would need to gather additional information about which of the following? A. Alcohol and caffeine consumption B. Client's workplace in relation to the surroundings C. Timing of symptoms in relation to the menstrual cycle D. Bathing frequency and living surroundings

ANS: C Rationale: Considering that the client has tender and painful breasts and often feels lumps within her breast tissue, it is most likely that she suffers from fibrocystic breast disease. To confirm these findings, the nurse should ask relevant questions about the characteristics and timing of symptoms in relation to the menstrual cycle. Symptoms of fibrocystic breast disease are noticeable before menstruation and usually abate during menstruation. The size of the cyst becomes larger before menstruation and often changes with the menstrual cycle. The nurse should further ask the client about her habits of smoking and consuming coffee, chocolate, and caffeinated soft drinks, not alcohol, because they aggravate the condition. Workplace surroundings or cleanliness habits do not matter because fibrocystic breast disease is not infectious.

18. A client is admitted to the hospital with the diagnosis of sepsis secondary to pelvic inflammatory disease. Which transmission-based precaution will be initiated by the nurse? A. Standard precautions B. Protective precautions C. Contact precautions D. Droplet precautions

ANS: C Rationale: Contact isolation is a category of transmission-based precautions for controlling the spread of infectious microorganisms found in body fluids. Standard precautions are used in the care of all clients in the prevention of HIV and hepatitis. Protective precautions are instituted when a client is immune compromised and needs protected from others. Droplet precautions are not indicated with PID.

7. The nurse is preparing a woman for an abdominal ultrasound. Which instruction would be appropriate for the nurse to include? A. Restrict solid food intake for 2 hours before the test. B. Refrain from douching for at least 1 week before the test. C. Drink at least 1 quart of water an hour before the test. D. Empty the bladder immediately before the test.

ANS: C Rationale: Drinking at least 1 quart of water 45 minutes to 1 hour before the test and not voiding until after the test ensures a full bladder and facilitates transmission of the ultrasound waves. It also elevates the bowel away from the other pelvic organs. The client should restrict solid food intake for 6 to 8 hours before the test to avoid having images of the test obscured by gas and intestinal contents. There is no restriction on douching for this test. A full bladder, not an empty one, facilitates this test.

31. The nurse is describing various terms related to the male reproductive system. Which term would the nurse use to describe the discharge of semen from the penis? A. Erection B. Emission C. Ejaculation D. Engorgement

ANS: C Rationale: Ejaculation refers to the discharge of semen from the penis. Erection refers to the state in which the penis becomes elongated and rigid. Dilation of the penile arteries compresses the veins within the penis causing engorgement of blood within the tissue. Emission refers to the movement of sperm and their mixture with fluid forms the seminal vesicles and prostate gland into the urethra.

5. Culture of client's vaginal discharge reveals Gardnerella vaginalis. Which of the following would the nurse expect to assess? A. Foul foamy discharge B. Thick curdy white discharge C. Fishy smelling watery discharge D. Yellowish white discharge

ANS: C Rationale: Gardnerella vaginalis is associated with a gray white, watery, fishy smelling vaginal discharge. The discharge associated with a Candida infection is curdy white and thick, and has a strong odor. Discharge due to trichomonas vaginalis is yellow white, foamy, and has a foul odor.

19. The nurse is relating health education to male students when asked where sperm is actually made. Which location is most correct? A. In the male reproductive system B. Sperm are present from birth C. In the testes within the scrotum D. In the seminiferous tubules

ANS: C Rationale: Health instruction should be clear, specific, and factual. The specific location for sperm production is in the testes, which lie in the scrotum. More general information is that the sperm are produced in the male reproductive system and immature spermatozoa are formed in the seminiferous tubules. Sperm are not present from birth.

20. A family member of a client with breast cancer asks the nurse how immunotherapy for cancer works. The nurse would be correct when making which statement? A. Immunotherapy strengthens the immune system to prevent cancer. B. Immunotherapy uses homeopathic properties to prevent free radical release. C. Immunotherapy stimulates the immune response to attack and rid the body of cancer cells. D. Immunotherapy promotes removal of good and bad multiplying cells.

ANS: C Rationale: Immunotherapy is the basis for various drugs and therapeutic cancer vaccines. Rather than prevent breast cancer, they stimulate the immune system's ability to attack and rid the body of cancer cells. Immunotherapy does not prevent cancer, prevent free radical release, and remove good and bad multiplying cells.

13. The nurse is preparing a client for a test that involves inserting a thick barium paste into the rectum with radiographs taken as the client expels the barium. What test will the nurse prepare the client for? A. Kidneys, ureters, bladder (KUB) B. Colonic transit studies C. Defecography D. Abdominal radiography

ANS: C Rationale: In defecography, a thick barium paste is inserted into the rectum. Radiographs are taken as the client expels the barium to determine whether there are any anatomic abnormalities or problems with the muscles surrounding the anal sphincter. A KUB will not determine this. Colonic transit studies are used to determine how long it takes for food to travel through the intestines. Abdominal radiography will show the structure but does not determine the muscle ability surrounding the anal sphincter.

22. The nurse is caring for a client experiencing metastatic cancer-related pain. Which drug of choice does the nurse anticipate being ordered? A. Ibuprofen (Motrin) B. Alprazolam (Xanax) C. Morphine (MS Contin) D. Propofol (Diprivan)

ANS: C Rationale: In treating clients with metastatic cancer-related pain, the opioid analgesics morphine (MS Contin) and fentanyl are the drugs most often used for pain relief. Morphine (MS Contin) can be given orally, rectally, subcutaneously, intravenously, intramuscularly, or by epidural catheter. Ibuprofen (Motrin) is given for mild to moderate pain. Alprazolam (Xanax) is provided to relieve anxiety. Propofol (Diprivan) is an anesthetic given frequently for short procedures.

5. A client realizes that regular use of laxatives has led to bowel pattern improvement. However, the nurse cautions this client against the prolonged use of laxatives for which reason? A. The client may develop inflammatory bowel disease. B. The client may develop arthritis or arthralgia. C. The client's natural bowel function may become sluggish. D. The client's appetite may be depleted.

ANS: C Rationale: It is essential for the nurse to caution the client against the prolonged use of laxatives because it decreases muscle tone in the large intestine. Prolonged use of laxatives may cause the client's natural bowel function to become sluggish. Laxatives do not increase the risk of developing inflammatory bowel disease, arthritis, or arthralgia, nor do they cause a loss in appetite.

21. The nurse is caring for a client who is employed as a manual laborer. The client has recovered from a recent mastectomy and is planning to return to work. Which instruction from the nurse would be most helpful? A. Do not return to work until all areas are completely healed. B. Have others assist in tasks involving use of affected arm. C. Wear gloves to prevent injury. D. Check to see if job description can be changed.

ANS: C Rationale: It is most helpful to provide an option that allows the client to continue employment at the current job, if the client is able. Many times, the client must continue employment and cannot change jobs. Instructing the client to not return to work may not be an option throughout the healing process, which could be up to 1 year. Asking others to do tasks and changing the job description are unrealistic throughout the recovery period.

30. The nurse is providing discharge teaching to a client following a vasectomy. Which is the most important instruction to provide? A. Wear a scrotal support until swelling is resolved. B. Use a Tylenol as needed for discomfort. C. Use another form of birth control until further notice. D. Take a day or two to rest and recuperate from the procedure.

ANS: C Rationale: It may take up to 10 ejaculations to clear sperm and prevent impregnation. The client should be instructed to use another form of reliable birth control until a sperm count proves sterility has occurred. Use of Tylenol, scrotal support, and rest are all helpful during the initial recovery period.

19. A client with a diagnosis of acute appendicitis is awaiting surgical intervention. The nurse listens to bowel sounds and hears none and observes that the abdomen is rigid and board-like. What complication does the nurse determine may be occurring at this time? A. Constipation B. Paralytic ileus C. Peritonitis D. Accumulation of gas

ANS: C Rationale: Lack of bowel motility typically accompanies peritonitis. The abdomen feels rigid and board-like as it distends with gas and intestinal contents. Bowel sounds typically are absent. The diagnosis of acute appendicitis correlates with the symptoms of rupture of the appendix and peritonitis. A paralytic ileus, constipation, and gas alone do not produce these symptoms.

17. The nurse is caring for a client with breast cancer and removal of axillary lymph nodes. Which assessment finding is documented and brought to the physician's attention as potential lymphedema? A. A reddened area around the breast B. Fluid accumulation under in the axilla C. Enlargement of the arm or hand D. Drainage from the areola

ANS: C Rationale: Lymphedema, soft-tissue swelling from accumulated lymphatic fluid, occurs in some women after they have undergone breast cancer surgery and the removal or irradiating of axillary lymph nodes. Lymphedema does not impact the breast area or axilla.

31. A client is diagnosed with prostatitis. Which is the most important teaching point for the management of this disorder? A. Avoid tub baths. B. Avoid sexual activity. C. Limit fluid intake. D. Avoid prolonged sitting.

ANS: D Rationale: Prolonged sitting can aggravate the condition of prostatitis and should be avoided. Warm tub baths and/or sitz baths can provide comfort. Increasing fluid intake can also flush the urethra of microorganisms and decrease risk of bacterial infection. Regularly draining the prostate gland through masturbation or intercourse can be helpful.

16. A client informs the nurse of having abdominal pain that is relieved when having a bowel movement. The health care provider diagnosed the client with irritable bowel syndrome. What does the nurse recognize as characteristic of this disorder? A. Weight loss due to malabsorption B. Blood and mucus in the stool C. Chronic constipation with sporadic bouts of diarrhea D. Client is awakened from sleep due to abdominal pain.

ANS: C Rationale: Most clients with irritable bowel syndrome (IBS) describe having chronic constipation with sporadic bouts of diarrhea. Some report the opposite pattern, although less commonly. Most clients experience various degrees of abdominal pain that defecation may relieve. Weight usually remains stable, indicating that when diarrhea occurs, malabsorption of nutrients does not accompany it. Stools may have mucus, but blood is not usually found because the bowel is not locally inflamed. The sleep is not disturbed from abdominal pain.

13. Following morning hygiene of an older adult client, the nurse is unable to replace the retracted foreskin of the penis. Which is the most likely outcome? A. Erection of the penis B. Unclean glans C. Painful swelling D. Nausea and vomiting

ANS: C Rationale: Paraphimosis results in strangulation of the glans penis from inability to replace the retracted foreskin. The strangulation results in painful swelling of the glans. Erection of the penis in the presence of phimosis can cause pain but is not a result of retracted foreskin. Nausea and vomiting are not indicated with retraction of foreskin.

26. Following a radical vulvectomy, the nurse is preparing the client for discharge to home. Which care intervention would be considered the priority for this client? A. Relieving edema to lower extremities B. Alterations for sexual function C. Prevention of wound complications D. Care of colostomy site

ANS: C Rationale: Prevention of complications and infection is paramount due to the location of the wound. The perineal area provides a warm, dark environment that supports the growth of microorganisms that can be introduced into the wound. With a radical vulvectomy, it is likely to find surgical drains and urinary catheter that can also interfere with the maintenance of the wound. Relieving edema to the lower extremities may be a necessary part of care if the lymph nodes and blood vessels are disturbed. The client may have a colostomy and care instructions should be provided. Alteration in sexual function needs to be addressed but is not a priority for the initial stage of healing.

7. A client who comes to the clinic complaining of perineal pain, dysuria, and fever is diagnosed with prostatitis. The nurse understands that which of the following organisms would be the most likely cause? A. Staphylococcus B. Streptococcus C. Escherichia coli D. Paramyxovirus

ANS: C Rationale: Prostatitis is an inflammation of the prostate gland and is most often caused by microorganisms that reach the prostate by way of the urethra. Escherichia coli and microbes that cause sexually transmitted infections often are responsible. Staphylococcus and Streptococcus are not typically a cause of prostatitis. Paramyxovirus is the cause of mumps.

16. The nurse is caring for a client whose physician has ordered a sentinel lymph node mapping. The physician explained the procedure and desired outcome. Which statement, made by the client, indicates a need for further instruction? A. The procedure allows for an understanding of the spread of cancer cells. B. The procedure allows for conservation of breast tissue. C. The procedure removes all cancer from the body. D. The procedure includes minimal surrounding tissue damage.

ANS: C Rationale: Sentinel lymph node mapping involves identifying the first (sentinel) lymph nodes through which the breast cancer cells would spread to regional lymph nodes in the axilla. Validating the lack of lymph node metastasis allows the surgeon to preserve more breast tissue, axillary tissue and chest muscle. Further instruction would be needed to explain that the sentinel lymph node biopsy does not remove cancer from the body.

3. A client with vaginitis complains of itching and burning of the perineum. Which suggestion by the nurse would be most appropriate to relieve the client's symptoms? A. Use a pure vinegar douche daily. B. Use skin protectants containing zinc oxide. C. Take sitz baths frequently. D. Avoid yogurt with active lactobacilli cultures.

ANS: C Rationale: Sitz baths are recommended to relieve the client's itching and burning as well as relieve swelling of the vulva and perineum. Skin protectants containing zinc oxide promote healing. Routine douching should be avoided. Taking Lactobacillus acidophilus in capsule form or eating yogurt containing active cultures of lactobacilli can help restore normal vaginal microorganisms.

8. The nurse is reviewing the laboratory test results of a client with Crohn disease. Which of the following would the nurse most likely find? A. Decreased white blood cell count B. Increased albumin levels C. Stool cultures negative for microorganisms or parasite D. Decreased erythrocyte sedimentation rate

ANS: C Rationale: Stool cultures fail to reveal an etiologic microorganism or parasite, but occult blood and white blood cells (WBCs) often are found in the stool. Results of blood studies indicate anemia from chronic blood loss and nutritional deficiencies. The WBC count and erythrocyte sedimentation rate may be elevated, confirming an inflammatory disorder. Serum protein and albumin levels may be low because of malnutrition.

21. A client with extensive endometriosis is scheduled for a panhysterectomy. Which statement by the client indicates a need for further teaching? A. "I will be having my uterus, tubes, and ovaries removed." B. "I am finished having children." C. "I will not have to deal with symptoms of menopause." D. "I will now have a greater risk for stroke and heart disease."

ANS: C Rationale: Surgical menopause causes a sudden drop in estrogen and progesterone levels resulting in varied symptoms in clients. The risks of heart disease and stroke increase with estrogen reduction. A panhysterectomy is the removal of the uterus, both tubes and ovaries and will result in the inability to conceive children.

4. A client is being admitted for suspected toxic shock syndrome (TSS). Which of the following would be most important for the nurse to determine? A. Existence of menorrhagia B. Psychological trauma C. Use of superabsorbent tampons D. Use of oral contraceptives

ANS: C Rationale: TSS is commonly associated with the use of superabsorbent tampons that are not changed frequently and internal contraceptives that remain in place longer than necessary. Assessing the use of oral contraceptives, psychological trauma, or menorrhagia is not required for diagnosing the cause of TSS.

5. The nurse has provided teaching to a client who is scheduled to have a Pap test. Which client statement indicates that teaching was successful? A. "I'll schedule the test for the time when I have my next menstrual period." B. "I need to refrain from sexual intercourse for at least 1 week before the test." C. "I will stop using any vaginal medications at least 48 hours before the test." D. "I need to use a vinegar and water douche the morning of the test."

ANS: C Rationale: Teaching is effective when the client schedules an appointment at a time other than during menstruation and does the following before the appointment: (1) avoids intercourse for 2 days, (2) refrains from douching for 1 day, and (3) ceases the use of vaginal medications for at least 48 hours.

14. A parent of an adolescent female is asking the nurse for guidelines of when the daughter should have her first Papanicolaou test (Pap test) and then the subsequent frequency. The nurse is most correct to state which? A. The first Pap test should be at age 16 years and then annually thereafter. B. The first Pap test should be no later than 18 years of age and then every other year afterward. C. The first Pap test should be between 25 and 29 years and then every 3 years afterward. D. The first Pap test should be within the first year of menstruation and annually afterward.

ANS: C Rationale: The American Cancer Society (ACS) recommends no testings/screenings for women aged 21 to 24 years. Women between the ages of 25 and 29 years have the recommendation for HPV testing to be every 5 years (preferred), the HPV/Pap co-test every 5 years (acceptable), or the Pap test every 3 years (acceptable).

10. The nurse is caring for a client who states having an irregular menstrual period. Which diagnostic test provides information of the body's ability to ovulate? A. Thyroid level B. Serum luteinizing hormone C. Serum follicle-stimulating hormone D. Estrogen level

ANS: C Rationale: The anterior pituitary hormone known a follicle-stimulating hormone (FSH) initiates ovulation monthly. Monitoring the FSH level provides information about the timing of ovulation. Thyroid level, estrogen level, and serum luteinizing hormone do not give information about the ability to ovulate

13. The nurse is obtaining a health history at the client's annual gynecologic exam. Which of the following statements regarding a Papanicolaou test (Pap test) for cancer screening would delay the exam? A. "I had sexual intercourse several days ago." B. "I voided before entering the exam room." C. "My menstrual period began yesterday." D. "I use no vaginal sprays or creams."

ANS: C Rationale: The nurse advises that the Pap test be rescheduled to a time other than during the menstrual period. The other options are acceptable to continue the test procedure.

6. The nurse is caring for a client who has just been diagnosed with endometriosis. The client has been hoping to have children with her partner. Which statement by the nurse is most appropriate to provide the client with support and guidance about treatment options? A. "Treatment is essential, so you really need to make a decision pretty quickly." B. "If it was me, I would probably choose the medication options." C. "It might help to include your partner in any of the discussions about options." D. "The test results are clear and another physician would tell you the same thing."

ANS: C Rationale: The nurse assists the client through the decision-making process as it applies to family planning and medical or surgical treatment. Suggesting that the client include a partner or significant other in the discussion of options would be most helpful and therapeutic. The client does not need to make a decision immediately. The nurse should not give advice or offer personal opinions to influence the client's choice. The nurse should support the client's option of seeking a second opinion.

9. The nurse is outlining the female internal reproductive structures on a diagram. Where on the diagram would the nurse highlight the typical site of ovum fertilization? A. The nurse would highlight the uterus. B. The nurse would highlight the cervix. C. The nurse would highlight the fallopian tube. D. The nurse would highlight the ovaries.

ANS: C Rationale: The nurse would highlight the fallopian tubes as the site of ovum fertilization. Once fertilized, the fertilized egg moves to the uterus for implantation. The cervix is the lower, narrowed neck portion leading to the center of the uterus. The ovaries release the ovum, which is swept into the fallopian tubes.

28. The nurse is completing the admission history for a client who is admitted for a reduction mammoplasty. Which of the following client statements is uncommon when explaining the rationale for the procedure? A. Back pain B. Low self-esteem C. Others disapprove D. Skin irritation

ANS: C Rationale: The rationale for a reduction mammoplasty most often comes from the client experiencing a complication due to the size of the breast. Back pain, low self- esteem, self-consciousness, and skin irritation are common rationales. The disapproval of others is not a common rationale.

13. The client asks the nurse if there is a pill that can be ordered to control the symptoms of menopause. Which assessment finding is most important in determining nursing care in association with hormone replacement therapy? A. Presence of kyphosis B. Symptoms of hot flashes C. Family history of breast cancer D. History of osteoporosis

ANS: C Rationale: The risk of endometrial or breast cancer in women prescribed HRT may outweigh the benefits of relieving symptoms of menopause and preventing kyphosis or hip fractures associated with osteoporosis.

25. The nurse is interviewing a postmenopausal client during an annual gynecologic exam. Which assessment finding presents the greatest concern in the care of this client? A. Positive for human papillomavirus (HPV) B. Use of pessary C. Vaginal bleeding D. Urinary stress incontinence

ANS: C Rationale: Vaginal bleeding in a client after menopause is abnormal and a predominant symptom for vaginal cancers. The incidence of vaginal cancer is higher among woman infected with HPV and those who use a pessary but neglect to remove and clean it. Urinary stress incontinence is not a significant concern.

2. While obtaining the history, the nurse learns the client's mother was treated with diethylstilbestrol (DES) during her pregnancy. The nurse determines that this client is at risk for which of the following? A. Vulvar cancer B. Breast cancer C. Vaginal cancer D. Endometrial cancer

ANS: C Rationale: Vaginal cancer is associated with the risk factor of being born to mothers treated with DES during their pregnancy. The upper posterior third of the vagina is the most common site of vaginal cancer. Metastatic lesions may occur in the cervix or adjacent areas such as the vulva, uterus, or rectum. DES is not a risk factor associated with vulvar or breast cancers. Endometrial cancer occurs in women who take estrogens without the addition of progesterone for 5 or more years during and after menopause.

18. The nurse observes the physician palpating the abdomen of a client that is suspected of having acute appendicitis. When the abdomen is pressed in the left lower quadrant the client complains of pain on the right side. What does the nurse understand this assessment technique is referred to? A. Referred pain B. Rebound pain C. Rovsing sign D. Cremasteric reflex

ANS: C Rationale: When an examiner deeply palpates the left lower abdominal quadrant and the client feels pain in the right lower quadrant, this is referred to as a positive Rovsing sign and suggests acute appendicitis. Referred pain indicates pain in another area but is not necessarily manipulated by the examiner. Rebound pain is indicated when the pain of palpation is worse when the pressure is off the site. The cremasteric reflex is a superficial reflex that is present in male clients.

13. The client arrives at a public health clinic worried that she has breast cancer since finding a lump in her breast. When the nurse assesses the client's breast, which assessment finding is characteristic of fibrocystic disease? A. One breast is larger than the other. B. The lump is firm and immovable. C. The lump is round and movable. D. Nipple retractions are noted.

ANS: C Rationale: When assessing a breast with fibrocystic disease, the lumps typically are different from cancerous lumps. The characteristic breast mass of fibrocystic disease is soft to firm, circular, movable, and unlikely to cause nipple retraction. A cancerous mass is typically irregular in shape, firm, and immovable. Lumps typically do not make one breast larger than the other. Nipple retractions are suggestive of cancerous masses.

6. A group of students are reviewing information related to the male reproductive system structures. The students demonstrate understanding of the information when they identify which structure as an accessory organ? A. Vas deferens B. Epididymis C. Scrotum D. Seminal vesicles

ANS: D Rationale: Accessory structures include the seminal vesicles, which join with the ductus deferens to become the ejaculatory duct, the prostate gland, and the bulbourethral glands. The vas deferens and epididymis are internal male reproductive structures. The scrotum is an external male reproductive structure.

6. After examination, a client is found to have a prostatic nodule and is scheduled for prostatic-specific antigen (PSA) testing. When the nurse is reviewing the results, which of the following would indicate that the nodule is malignant? A. 4 ng/mL B. 7 ng/mL C. 9 ng/mL D. 12 ng/mL

ANS: D Rationale: After a prostatic nodule is detected, prostate-specific antigen testing may be done. A PSA greater than 4 ng/mL is the basis for performing more definitive diagnostic procedures, and a PSA greater than 10 ng/mL indicates a prostatic malignancy. A PSA greater than 80 ng/mL indicates advanced metastatic disease.

21. Which pharmacologic agents pose the greatest risk for urinary retention in a client with benign prostatic hyperplasia (BPH)? A. Muscle relaxers B. Antihypertensives C. Nitrates D. Antihistamines

ANS: D Rationale: Antihistamines and over-the-counter cold medications should be avoided in clients with BPH due to the increase in urinary retention properties. Muscle relaxers are commonly prescribed for treatment of urinary retention. Nitrates and antihypertensive medications do not cause significant risk in the management of BPH.

17. A client is recently diagnosed with Crohn disease and is beginning treatment. What first-line treatment does the nurse expect that the client will be placed on to decrease the inflammatory response? A. Ciprofloxacin B. Methotrexate C. Azathioprine D. Sulfasalazine

ANS: D Rationale: Considered first-line treatment for inflammatory bowel disease, 5-ASA drugs contain salicylate, which is bonded to a carrying agent that allows the drug to be absorbed in the intestine. These drugs work by decreasing the inflammatory response. Methotrexate or azathioprine are used when failure to maintain remission necessitates the use of an immune-modulating agent. Ciprofloxacin is used as an effective adjunct to treat the disease.

10. A client is complaining of problems with constipation. What dietary suggestion can the nurse inform the client may help facilitate the passage of stool? A. Increase the carbohydrate content of the diet. B. Increase dietary fat consumption. C. Increase dietary protein such as lean meats. D. Increase dietary fiber.

ANS: D Rationale: Constipation may result from insufficient dietary fiber and water. A diet low in fiber predisposes people to constipation because the stools produced are small in volume and dry. Increasing the carbohydrate, fat, and protein content will not facilitate the passage of stool.

27. A young client has just completed the 6-month series of Gardasil vaccine. Which comment by the client would alert the nurse that further teaching is needed? A. "This vaccine lowers my risk for cervical cancer." B. "I will still need to have routine screening for cervical cancer." C. "I am at lower risk for developing cervical warts." D. "This vaccine will prevent human papilloma virus (HPV) from occurring."

ANS: D Rationale: Gardasil is a vaccine that protects against four types of HPV, which cause 70% of cervical cancers and 90% of cervical warts but does not protect against other strains of HPV and pre vaccination infections. Routine cervical cancer screening is recommended regardless of vaccine use.

1. The instructor is teaching a group of students about intestinal obstruction. The instructor determines that the teaching was effective when the students identify which of the following as a cause of a functional obstruction? A. Volvulus B. Intussusception C. Tumor D. Abdominal surgery

ANS: D Rationale: In functional obstruction, the intestine can become adynamic from an absence of normal nerve stimulation to intestinal muscle fibers. For example, abdominal surgery can lead to paralytic ileus. Mechanical obstructions result from a narrowing of the bowel lumen with or without a space-occupying mass. A mass may include a tumor, adhesions (fibrous bands that constrict tissue), incarcerated or strangulated hernias, volvulus (kinking of a portion of intestine), intussusception (telescoping of one part of the intestine into an adjacent part), or impacted feces or barium.

29. The nurse is preparing a client for a pelvic examination. Which nursing question is essential for the nurse to ask? A. "Are you sexually active?" B. "Do you have any sexually transmitted diseases?" C. "Would you like to have assistance to get in position for the exam?" D. "Would you like to void at this time?"

ANS: D Rationale: Prior to a pelvic examination, the nurse offers the client the use of the restroom to void. It is most important for the client to empty the bladder so that the physician can feel pelvic structures more clearly and for the comfort of the client. Asking client history questions is completed at the beginning of the appointment. It is important to offer assistance to those who may need help in assuming the lithotomy position.

12. A client at a long-term care facility informs the nurse that he is having cramping when trying to have a bowel movement, and all that is coming out is liquid. When the nurse reviews the client's last bowel movement history, it is determined that the client has not had a bowel movement in 7 days. What does the nurse understand is most likely occurring with this client? A. Scybala B. The history is incorrect of the last bowel movement. C. Diarrhea D. Encopresis

ANS: D Rationale: Sometimes, if a client has been constipated for a long time, the client may begin passing liquid stool around an obstructive stool mass called encopresis, a phenomenon sometimes misinterpreted as diarrhea. The liquid stool results from dry stool stimulating nerve endings in the lower colon and rectum, which increases peristalsis. Scybala is hard, rocklike stool. The nurse cannot make a judgment about the correctness of the last bowel movement if it is not documented. Encopresis will mimic diarrhea, but there is an obstructive mass above where the liquid stool is leaking around.

26. The nurse is caring for a client following breast reconstruction surgery using autogenous tissue. When assessing wounds from the surgical procedure, in which area would the nurse assess the wound from which tissue was taken? A. The vastus lateralis B. The rectus femoris C. Gluteus maximus D. Rectus abdominis

ANS: D Rationale: The area in which tissue is taken for breast reconstruction surgery is from the rectus abdominis muscle in a manner similar to a "tummy tuck." The nurse must assess this site for healing or complications.

5. A female client is diagnosed with breast abscess. She would like to continue to breast-feed her newborn. Which action by the nurse would be most appropriate in this situation? A. Encourage the client to include protein content in the diet. B. Instruct the client to wear a tight-fitting bra. C. Reduce the frequency of removing and reapplying the dressings. D. Assist the client to pump the breasts to remove breast milk.

ANS: D Rationale: The nurse should help the client pump the breasts and remove breast milk to prevent engorgement. Because the client has decided to continue breastfeeding, the client should wear a loose-fitting bra. Including protein content in the diet would be unrelated to the client's current situation. Frequency of dressing changes does not play a role in the intervention.

4. Which of the following should nurses teach all men, especially those who have had cryptorchidism? A. Need for blood tests to measure serum acid phosphatase levels B. Importance of regular monitoring of prostate-specific antigen (PSA) levels C. Need to undergo a baseline and follow up lymph node biopsies D. How to perform a testicular self-examination

ANS: D Rationale: The nurses should teach all men, especially those who have had cryptorchidism, to perform testicular self-examination to detect any abnormal mass in the scrotum. The nurse instructs the clients to examine the testicles monthly, preferably when warm, such as in the shower. Having regular PSA levels, lymph node biopsies, and blood tests for measuring serum acid phosphatase are for the clients who are treated for prostate cancer.

12. The nurse is conducting a health history when a middle-aged client reports last having a menstrual period 6 months ago. Upon further questioning, the client also states symptoms of hot flashes and mood fluctuations. Which question should the nurse ask next? A. "Do you feel like hurting yourself?" B. "Are you finished having children?" C. "When was your first menstrual period?" D. "Are you taking any hormone replacement therapy?"

ANS: D Rationale: To ensure a thorough health history, a client who exhibits symptoms of perimenopause should be assessed for the use of hormone replacement therapy to alleviate the symptoms. This information adds to the data reported by the client. Asking if the client is having thoughts of self-harm may be extreme with the report of mood fluctuations. Asking if the client is finished having children produces little additional data. Asking the first menstrual period is part of the health history but not the best question to ask after the client's statement.

7. A client with a hernia decides to manage the herniation with a truss. The nurse would emphasize which of the following? A. Using laxatives to ensure regular bowel movement B. Wearing warm, woolen clothes to avoid dryness C. Applying a sunscreen to prevent exposure to direct sunlight D. Using cornstarch to absorb moisture in the area

ANS: DRationale: When a client is managing herniation with a truss, the nurse informs the client to keep the skin clean and dry or to use cornstarch to absorb moisture. This minimizes the risk for infection. Use of warm, woolen clothes will not help reduce moisture, it may increase the moisture and increase the risk of infections. If the client's bowel movements are regular, laxatives would not be necessary. However, the client would need teaching to prevent constipation. Applying sunscreen is a general recommendation for any client to reduce the risk of exposure to ultraviolet radiation from the sun.


Conjuntos de estudio relacionados

EA Unit 9: Rental and Royalty Income

View Set

NU141- Chapter 54 Drugs Acting on the Upper Respiratory Tract

View Set